Discussion

The physicist’s reasoning is most vulnerable to criticism on which one of the following grounds?
(A)That it is impossible to measure accurately both the position and velocity of any given subatomic particle does not imply that it is impossible to know either the position or velocity of all subatomic particles.
(B)...
(C)...
(D)...
(E)...
(F)...
*This question is included in Practice Set: "Flawed Reasoning" Questions, Set 3 - Mix (25Q), question #22

The solution is

Posted: 08/08/2011 04:49
I don't quite understand why the answer is (c) but not (b). Anybody can help?
Posted: 08/08/2011 19:16
This is an "EXCEPT" question, so you're looking for the answer choice that does NOT describe a flaw in the argument's reasoning.

Let's take a look at the argument:

Premise 1: 90% of "I" are also "CD"
Conclusion: Tom: "CD" --> Tom: "I"

Note that this argument assumes that, in general, CD --> I.

Now, on to the answer choices:
Choice "B" says one of the argument's flaws is that it fails to consider the possibility that things other than coffee cause insomnia.

The argument implies that since most insomniacs are heavy coffee drinkers, coffee drinking likely causes insomnia in Tom. That is, the argument assumes that a correlation between coffee drinking and insomnia implies that coffee causes insomnia. So the argument DOES fail to consider that something else could be the cause of the insomnia in coffee drinkers. This makes "B" wrong.


Choice "C":
This choice basically says that the argument is flawed because it tells you what percentage of insomniacs drink a lot of coffee, but it doesn't tell you what percentage of coffee drinkers have insomnia.

Thing is, even if it gave this information, you STILL couldn't jump to the conclusion that since Tom drinks a lot of coffee, he must be an insomniac. If you did that, you'd still be confusing correlation for causation. So choice "C" is NOT a flaw in the argument, and it is the correct answer.

Posted: 12/11/2011 15:58
I'm pretty sure it's A. The argument doesn't say that 90 percent of coffee drinkers are insomniacs; it says 90% of insomniacs are coffee drinkers.
So to say it fails to consider that he's among the 10 percent of coffee drinkers who aren't insomniacs presumes that the argument makes the former point, when it of course does not..
Posted: 12/11/2011 16:01
Also just because had C been satisfied, the argument might still be flawed does not show that C is not a flaw.
Image Not Available
Contributor
Posted: 12/11/2011 22:06
Adam,

This question asks you to find the answer choice that does NOT describe a flaw in the argument.

You seem to be arguing that choice (A) is flawed, which it is. So are choices (B), (D), and (E).

Remember, the question stem says:
"Each of the following describes a flaw in the argument's reasoning EXCEPT..."
Posted: 12/12/2011 01:28
The correct answer should be a flawed flaw, which A is. C is a correct flaw. It is something the arguer didn't consider but should have considered, as are B, D, and E. A makes no sense, so it's not a correct description of a flaw in the argument. Rather it's the answer which doesn't pertain at all...
What am I missing?
Posted: 12/12/2011 01:33
C would be the correct answer if the prompt was "which of the following best describes the central flaw in the argument?"
Posted: 12/12/2011 01:41
In fact C is really the only serious flaw.
A is just the most ridiculous, which is why I chose it...
Posted: 12/12/2011 12:32
I will humbly submit that this question has the wrong prompt. C is the only real flaw in the argument; it's certainly the clearest flaw, which makes it seemingly hilarious as the proposed correct answer for the prompt as it reads currently...
Posted: 12/12/2011 13:21
I would agree with Adam. The argument contains a flaw of "Mistaken Reversal," as follows:
Premise: "EI" (mostly) ->"DC", but Conclusion: "DC"(mostly)->"EI".
C correctly points out that there is no such premise supporting: "DC" (mostly) ->"EI". C therefore describes a flaw in the argument, so cannot be the correct answer.
Image Not Available
Contributor
Posted: 12/12/2011 23:47
We looked at our original files again (sent to us by LSAC). The file is an rtf that was clearly OCR'd (created using optical character recognition software on an old copy of the actual LSAT test). The original question in our copy does contain the "EXCEPT" stem.

However, we think this is an error, as the file is rife with errors (including, in some cases, question stems that were omitted all together, rendering the questions unusable).


After reading through the argument and answer choices again, I think that both Adam W and J Yeo are correct. I think that the stem must be mismatched, because the flaw is definitely described in choice "C." Additionally, the other answer choices describe flawed thinking, just not the flawed thinking used to draw the conclusion in the argument.

I'd like to apologize for any confusion, and also for being so trusting of the source material.

Way to stick to you guns Adam W and J Yeo. Keep up the good work.


We'll sub a new question in for this one ASAP. Look for it on the next update.

You need to be signed in to perform that action.

Sign In